54 x 8181 x 12 x 8271 x 1301 x 812

Answers

Answer 1

Answer:

4.6320447e+16

I used a calculator for this LOL.


Related Questions

PLEASE HELP!! Need by 3/29/22!
Find the area of these shapes

Answers

Answer:

Step-by-step explanation:

area of A=15*18=270 cm^2

area of B=15*18=270 cm^2

area of C=1/2 ×x×18=9x cm²

area of D=1/2×15×18=135 cm²

Kindly solve this question

Answers

Answer:

x=-1/7

Step-by-step explanation:

[tex]\frac{3x-1}{3} -\frac{2x}{x-1} =x[/tex]

[tex]\frac{(3x-1)(x-1)}{3x-3} -\frac{2x(3)}{3x-3} =x[/tex]

[tex]\frac{3x^2-4x+1}{3x-3} -\frac{6x}{3x-3} =x[/tex]

[tex]\frac{3x^2-10x+1}{3x-3} =x[/tex]

[tex]\frac{3x^2-10x+1}{3x-3} =\frac{x(3x-3)}{3x-3}[/tex]

[tex]\frac{3x^2-10x+1}{3x-3} =\frac{3x^2-3x}{3x-3}[/tex]

[tex]3x^2-10x+1=3x^2-3x[/tex]

[tex]-10x+1=-3x[/tex]

[tex]-7x=1[/tex]

[tex]x=-1/7[/tex]

What is the approximate measure of angle B?

Answers

Answer: D

Step-by-step explanation:

HELP PLEASE!! <3
i honestly don't understand what they want me to do here

Answers

Answer:

(a)

Step-by-step explanation:

the equation of a circle centred at the origin is

x² + y² = r² ( r is the radius )

x² + y² = 20 ← is in this form , then

centre = (0, 0 ) and r = [tex]\sqrt{20}[/tex] = [tex]\sqrt{4(5)}[/tex] = [tex]\sqrt{4}[/tex] × [tex]\sqrt{5}[/tex] = 2[tex]\sqrt{5}[/tex]

What is the area of the trapezoid?



Enter your answer in the box.

Answers

378 inches squared

its (18x15)+(6x18)

A farmer plans to build the small silo shown to store chicken feed. What is the circumference of the base of the silo if it can hold a maximum of 132 cubic feet of feed?

Answers

The circumference of the base of the silo that can hold a maximum of 132 cubic feet of feed is approximately 28.796 feet.

How to determine the circumference of the base of a silo

The circumference is the measure of the contour of a circle. In this question we have to determine the radio of the base of the cone from the volume formula and later the circumference is found by this finding:

Volume

V = (π/3) · R² · h     (1)

Circumference

s = 2π · R     (2)

Where:

R - Radius of the base, in feeth - Height of the silo, in feet

If we know that V = 132 ft³, h = 6 ft, then the circumference that can hold the given capacity is:

[tex]R = \sqrt{\frac{3\cdot V}{\pi \cdot h} }[/tex]    

[tex]R = \sqrt{\frac{3\cdot (132\,ft^{3})}{\pi\cdot (6\,ft)} }[/tex]

R ≈ 4.583 ft

s = 2π · (4.583 ft)

s ≈ 28.796 ft

The circumference of the base of the silo that can hold a maximum of 132 cubic feet of feed is approximately 28.796 feet. [tex]\blacksquare[/tex]

Remark

The statement is complete, complete form is introduced below:

A farmer plans to buid the small silo  to store chicken feed. The silo is a cone with a height of 6 feet. What is the circumference of the base of the silo if it can hold a maximum of 132 cubic feet of feed?

To learn more on volumes, we kindly invite to check this verified question: https://brainly.com/question/1578538

HELPPPPPPPPPPPPPPPPPPPPPP

Answers

Answer:

x=6

Step-by-step explanation:

i hope it's helpful for you

Answer:

x=6

Step-by-step explanation:

24÷4=6

4×6=24

Hope it helps :))

Charmaine got a prepaid debit card with $15 on it. for her first purchase with the card, she bought some bulk ribbon at a craft store. the price of the ribbon was 21 cents per yard. if after that purchase there was $12.27 left on the card, how many yards of ribbon did charmaine buy?

Answers

Answer:

15- 12.27 = 2.73

2.73 is how much she spent on the ribbon.

2.73/ 0.21 = 13

She purchased 13 yards.

Photo attached with instructions

Answers

Step-by-step explanation:

using cosine rule,

BC²= AC² + BC² - 2*AC*BC*COSc

= 13² + 6² - 2*13*6*-cos(180-91)

= 169 + 36 - (-2.72)

= 202.28

√BC² = √202.28

BC = 14.22

BC = 14.20cm to nearest tenth

What is the value of w in the equation 2(2w − 16) = 0?

Answers

Answer:

[tex]w=8[/tex]

Step-by-step explanation:

[tex]2(2w-16)=0\\4w-32=0\\4w=32\\w=8[/tex]

Answer:

w = 8

Step-by-step explanation:

2(2w − 16) = 0

Divide each side by 2

2w-16 = 0

Add 16 to each side

2w-16 +16 =0 +16

2w = 16

Divide each side by 2

2w/2 = 16/2

w = 8

Please help! will give brainlest! locks in 30 min!

Answers

Answer:

[tex]\frac{\sqrt{2} }{2}[/tex]

Step-by-step explanation:

Simplify the expression:

sin [tex]\frac{3pi}{4}[/tex]

Rewrite the angle as a sum or difference with its reference angle:

sin (pi - [tex]\frac{pi}{4}[/tex])

Simplify the expression

(sin [tex]\frac{pi}{4}[/tex])

Evaluate the expression:

[tex]\frac{\sqrt{2} }{2}[/tex]

hope this helps and is right!! p.s. i really need brainliest :)

Simplify: 14-30/ 2 (- 4)

Thanks! :)

Answers

Answer:

The answer is 2

-16/-8=2

A square rug has an area of 256 square feet. What is the perimeter of the rug?

Answers

Answer:

64

Step-by-step explanation:

[tex]\sqrt{256}[/tex] = 16 (length of each side)

A square (rug) has 4 sides.

Add to find perimeter:

16 + 16 + 16 + 16

= 64

Answer: 8ft

Step-by-step explanation:

4A= 256

4A/4     =    256/4

A = 64

A = a²

64 = 4²

a² = 64

a = ±√64

a = ±8

∴a = ±8

Hence, the length of a side of each rug is 8ft.

Solve for area and perimeter in each of the following problems.

Answers

Answer:

1. perimeter =2(l+w)

                    =2(17+10)

                   = 54 yards

Are of the perimeter=l times b

                                =17 x 10

                                =170 yd square

Just follow this concept for all of them. It the same thing. Good Luck!!

Step-by-step explanation:

which is the answers

Answers

Answer:

1. any number from 42 to 68

e.g 52

2. any number less than 42 or greater than 68

e.g 76

What is the tangent ratio of angle KJL?

Answers

Answer: (1)

Step-by-step explanation:

Using the properties of a 30 60 90 triangle, we get that ML=24sqrt3 and JM=24.

This means that the tangent is 24sqrt3 / 24.

The ratio of the number of dolls Jacky had to the number of dolls Peter had was 5:2 but, after Jacky gave 15 dolls to Peter, they have an equal number of dolls. How many dolls did they have altogether?​

Answers

Answer:

70 dolls

Step-by-step explanation:

Hello!

The ratio is 5:2, which are division of  a whole. They can be represented as 5x and 2x.

Jacky has 5x dolls, and Peter has 2x dolls. They were equal after subtracting 15 dolls from 5x and adding them to 2x.

Equation:

5x - 15 = 2x + 15

Solve:

5x - 15 = 2x + 155x - 30 = 2x5x = 2x + 303x = 30x = 10

So Jacky has 5(10) dolls, or 50 dolls, and Peter has 2(10) or 20 dolls.

The total sum is 70 dolls.

You have a cup of lemonade. Each ice cube has a length of 2 cm, a width of 1 1/4 cm and a height of 3 cm. What is the volume of one of the ice cubes? Show your steps and Explain how you used math to come to your decision.

Answers

volume : 7.5 cm²

volume of cube : length * width *  height

[tex]\hookrightarrow \sf Length \ * Width \ * \ Height[/tex]

[tex]\hookrightarrow \sf 2 \ * 1\dfrac{1}{4} \ * \ 3[/tex]

[tex]\hookrightarrow \sf 2 \ * \dfrac{5}{4} \ * \ 3[/tex]

[tex]\hookrightarrow \sf \dfrac{15}{2}[/tex]

[tex]\hookrightarrow \sf 7.5 \ cm^2[/tex]

Answer:

[tex]\sf volume=7 \frac12 \ cm^3[/tex]

Step-by-step explanation:

Formula

volume of cuboid = length × width × height

Given dimensions of the ice cube:

length = 2 cmwidth = 1 1/4 cmheight = 3cm

Substituting the given values into the formula:

[tex]\sf \implies volume=2\times 1 \frac14 \times 3[/tex]

Converting to improper fractions:

[tex]\sf \implies volume=\dfrac21\times \dfrac54 \times \dfrac31[/tex]

                  [tex]\sf =\dfrac{ 2\times 5 \times 3}{1 \times 4 \times 1}[/tex]

                  [tex]\sf =\dfrac{30}{4}[/tex]

Dividing the numerator and denominator by the highest common factor:

[tex]\sf \implies volume=\dfrac{30 \div 2}{4 \div 2}[/tex]

                  [tex]\sf =\dfrac{15}{2}[/tex]

Converting the improper fraction into a mixed number:

[tex]\sf \implies volume=7 \frac12 \ cm^3[/tex]

Triangle ABC is congruent to B triangle A"B"C".
Which sequence of transformations could have been used to transform triangle abc to produce triangle A'' B'' C''? transform triangle ABC triangle A"B"C"?
A. Triangle ABC was translated 7 units down and then 9 units right.
B. Triangle ABC was translated 10 units to the right and then reflected across the x Axis.
C. Triangle ABC was reflected across the x Axis and then translated 9 units right.
D. triangle ABC was reflected across the y Axis and then translated 7 units down. ​

Answers

Answer: D

Step-by-step explanation:


A. where will vertex A be located after a rotation of -90°

B. where will vertex B be located after a rotation of 90°

C. where will vertex C be located after a -180° rotation

D. where will vertex D be located after a 270° rotation?

Answers

Answer:

this image should help a lot:

vertex b: (-5,3)

vertex a: (0,-1)

vertex c: (-6,-7)

vertex d: (-4,-1)

Step-by-step explanation:

log2^2-log2^16 what is the answer

Answers

Answer:

-4.1-4.1212

Step-by-step explanation:

Log2^2-log 2^16

log2^2/2^16

log2^2-16

log2^-14

-14log2

-14×0.3010

-4.1-4.1214

on a coordinate plane, the endpoints of line segment JL are
J(-6,-4) and L(6,8). Point K lies on line segment JL and divides it into two
line segments such that the ratio of JK to KL is 5:1. What are the
coordinates of point K?*

Answers

Answer: (4,6)

Step-by-step explanation:

We are splitting the line segment into 6 parts, so we can start by finding the displacement in the x and y coordinates.

The change in x is 12, and the change in y is 12.

This means that for each of the 6 parts we are splitting the segment into, the value of x will change by 2, and so will y.

This means that to find K, we can subtract one of these displacements from point L.

This means that K has coordinates (6-2, 8-2)=(4,6)

Please help and explain WILL MARK BRAINLIEST!!!!

Answers

Answer:

x is 19.5 degress

Step-by-step explanation:

hmmm so here oh i see now okay:

since a line is 180 degress lets write

2x + 82.5 + 3x = 180

now we umm add the ones with the same variable and power

5x + 82.5 = 180

5x = 180 - 82.5

5x = 97.5

x = 19.5

This is an angle on a straight line and It is equal to 180°

2x° + 3x° + 82.5° = 180°

5x° = 180° - 82.5°

5x = 97.5°

x = 19.5°

To indicate the angle

2x => 2(19.5°) => 39°

3x => 3(19.5°) => 58.5°

The angle are (39°, 82.5°, 58.5°

Least common multiple of 45 and 7

Answers

Answer:

315

Step-by-step explanation:

45 x 7 = 315

Multiples of 7: 7, 14, 21, 28... 301, 308, 315

Multiples of 45: 45, 90, 135, 180, 225, 270, 315, 360, 405

If you need further explanation, let me know!

- profparis

Step-by-step explanation:

Least common multiple of 45 and 7 is 315

gi i need help with this question if you can help me asap that whould be great​

Answers

It is 20 pluse 2 _hdjeihee

can anyone help me with this

Answers

Answer:

Yes

Step-by-step explanation:

Their angles are the same and the shapes are similar.

Answer:

Yes they do.

Step-by-step explanation:

All of the angles are the same shape and size therefore are equal to each other.

What information is included in this loan agreement that would NOT be included if Ron had gotten a loan from a bank or credit union?

Answers

Ron Is broke I think

Demarcus has to wrap a gift for his friend's birthday party. The gift is in a rectangular box with the dimensions shown below. He's trying to figure out how much gift wrap he needs to cover the entire box. Should he calculate the total surface area or the lateral surface area? Pick the right measurement and then calculate it for him.

In your answer, specify whether he should use total surface area or lateral surface area. Calculate the one you choose, give that measurement of the box, and then explain how you calculated it.

Answers

The surface area of the box is 1048 square inches

How to calculate the total surface area of the box

The formula for calculating the surface area of the prism is expressed as:

Surface area = 2(lw + wh +lh)

Given the following

l = 20in

w = 8in

h = 13in

Substitute the given parameters'

Surface area = 2(lw + wh +lh)

Surface area = 2(20(8) + 8(13) +20(13))

Surface area = 2(160+104+260)

Surface area = 1048 square inches

Hence surface area of the box is 1048 square inches

Learn more on surface area of box here: https://brainly.com/question/26161002

Use the following compound interest formula to complete the problem. a = p (1 startfraction r over n endfraction) superscript n superscript t victor has a credit card with an apr of 13.66%, compounded monthly. he currently owes a balance of $1,349.34. assuming that victor makes no purchases or payments, how much will he owe after one year, to the nearest cent? a. $1,349.34 b. $1,533.66 c. $1,545.65 d. $1,364.70

Answers

The final amount that Victor will owe after one year, to the nearest cent is given by: Option C: $545.65

How to find the compound interest?

If n is the number of times the interested is compounded each year, and 'r' is the rate of compound interest annually, then the final amount after 't' years would be:

[tex]a = p(1 + \dfrac{r}{n})^{nt}[/tex]

For this case, we're provided that:

The interest rate is r = 13.66% = 13.66/100 = 0.1366 (converted percent to decimal)

It is compounding monthly, thus, 12 times a year, or n = 12

The initial amount that the credit card of Victor has = p = $1349.34

Time for which interest was compounded = a year = 1 = t

Thus, the final amount that Victor will owe after one year to the nearest cent is calculated as;

[tex]a = p(1 + \dfrac{r}{n})^{nt}\\\\a = 1349.34(1 + \dfrac{0.1366}{12})^{12\times 1}\\\\a = 1349.34(1.01138)^{12} \approx 1245.65 \: \rm (in \: dollars)[/tex]

Thus, the final amount that Victor will owe after one year, to the nearest cent is given by: Option C: $545.65

Learn more about compound interest here:

https://brainly.com/question/1329401

Answer:

C

Step-by-step explanation:

I took the test and passed with 100

If 5^2x = 1, what is the value of 'x' ?​

Answers

Answer:

x = 0

Step-by-step explanation:

5^0 is 1, meaning that x must equal to 0 for the answer to be 5^0.

X = 0.04 because 5^2 (0) = 0 not 1
Other Questions
Which particle model diagram represents xenon at stp? Can someone please help with number 37 How many judges did trump appoint to the supreme court. A tower that is 166 feet tall casts a shadow 167 feet long. Find the angle of elevation of the sun to thenearest degree. Determine whether the following statement is true or false. If false, provide a counterexample. If two events are independent, then the probability of both events is less than 1 Write the balanced NET ionic equation for the reaction when ZnI and Pb(ClO) are mixed in an aqueous solution. A 50kg ball rolls off a building it is going 5 m/s when it hits the ground. How tall is the building Let g be the function given by g(x)=limh0sin(x h)sinxh. What is the instantaneous rate of change of g with respect to x at x=3? What is the difference between Spanish and Spanish III classes? need help with This no links or I will report you Help me, please don't answer if you don't know. Find the volume of thefollowing triangular prism.height: 2cmlength: 5cmwidth: 3.5cm A. 17.5cm3B. 35cm3C. 42.5cm3D. 45cm3 1.According to the evidence, what do you think is Ariadnes biggest character flaw? Be specific and explain.2.Theseus ship has vanished over the horizon and Ariadne has been left, alone on the island. What thoughts do you think would be going through her head as she rethinks her own behavior?3. Explain how/why Theseus would be considered a complex character? Support your answer using text evidence4. Which one of these characters reminds you most of yourself- King Aegeus, Theseus, King Minos or Ariadne? Tell me which one and explain in what way the character reminds you of yourself. This figure represents a ramp for a dollhouse that is being covered with fabric on all sides. How much fabric is needed to cover the ramp? 144 cm 138 cm 110 cm 66 cm What is one of the oldest religious practices? Communion Festivals Incense Prayer A boy stands 190m from a high wall and claps his hands,if he hears an echo 1.3 seconds later. calculate the speed of sound in air. Help pls!!!!!!!!!ddddddddddddddd The Pythagorean theorem states that a + b = c for a right triangle with leg lengths, a and b, and hypotenuse length, c.The hypotenuse of a right triangle is 5 units long and has the points (3, 0) and (0, 4) as end points. One of the legs has length 3.Use the Point and Segment tools to draw a right triangle at demonstrates the other leg length is 4. Please help with an explanation of how you found the answer because I am really confused. I will mark brainliest for best answer! The number 2 is a factor of _________.A) all odd numbersB) all even numbersC) all whole numbers